Menu Close

Author: Tinku Tara

k-0-n-cos-a-kb-

Question Number 72483 by TawaTawa last updated on 29/Oct/19 $$\underset{\mathrm{k}\:=\:\mathrm{0}} {\overset{\mathrm{n}} {\sum}}\:\mathrm{cos}\left(\mathrm{a}\:+\:\mathrm{kb}\right) \\ $$ Answered by Tanmay chaudhury last updated on 29/Oct/19 $${T}_{{k}} ={cos}\left({a}+{kb}\right) \\…

Find-all-continuous-functions-f-x-such-that-f-2x-1-f-x-for-all-real-x-

Question Number 6946 by 314159 last updated on 03/Aug/16 $${Find}\:{all}\:{continuous}\:{functions}\:{f}\left({x}\right)\:{such}\: \\ $$$${that}\:{f}\left(\mathrm{2}{x}+\mathrm{1}\right)={f}\left({x}\right)\:{for}\:{all}\:{real}\:{x}. \\ $$ Commented by Yozzii last updated on 04/Aug/16 $${f}\left(\mathrm{1}\right)={f}\left(\mathrm{0}\right) \\ $$$${f}'\left({x}\right)=\mathrm{2}{f}'\left(\mathrm{2}{x}+\mathrm{1}\right) \\…

x-2-3-x-1-dx-

Question Number 6945 by Tawakalitu. last updated on 03/Aug/16 $$\int\:\frac{{x}^{\frac{\mathrm{2}}{\mathrm{3}}} }{{x}\:+\:\mathrm{1}}\:\:{dx} \\ $$ Commented by Yozzii last updated on 03/Aug/16 $${x}={u}^{\mathrm{3}} \Rightarrow{dx}=\mathrm{3}{u}^{\mathrm{2}} {du} \\ $$$${x}^{\mathrm{2}/\mathrm{3}}…

tan-3-x-sec-3-x-dx-

Question Number 138009 by bobhans last updated on 09/Apr/21 $$\int\:\mathrm{tan}\:^{\mathrm{3}} \left({x}\right)\:\sqrt{\mathrm{sec}\:^{\mathrm{3}} \left({x}\right)}\:{dx}\:=? \\ $$ Answered by EDWIN88 last updated on 09/Apr/21 $$\mathcal{E}\:=\:\int\:\mathrm{tan}\:^{\mathrm{3}} \left({x}\right)\:\mathrm{sec}\:\left({x}\right)\:\sqrt{\mathrm{sec}\:\left({x}\right)}\:{dx} \\ $$$$=\:\int\:\mathrm{tan}\:\left({x}\right)\mathrm{sec}\:\left({x}\right)\left(\mathrm{sec}\:^{\mathrm{2}}…

please-solve-L-but-do-not-use-L-Hopital-s-rule-L-lim-x-0-1-x-1-e-x-1-

Question Number 6939 by sou1618 last updated on 03/Aug/16 $$\mathrm{please}\:\mathrm{solve}\:{L} \\ $$$$\mathrm{but}\:\mathrm{do}\:\mathrm{not}\:\mathrm{use}\:\mathrm{L}'\mathrm{Hopital}'\mathrm{s}\:\mathrm{rule}. \\ $$$${L}=\underset{{x}\rightarrow\mathrm{0}} {\mathrm{lim}}\left(\frac{\mathrm{1}}{{x}}−\frac{\mathrm{1}}{{e}^{{x}} −\mathrm{1}}\right) \\ $$ Commented by Yozzii last updated on 03/Aug/16…